LSAT and Law School Admissions Forum

Get expert LSAT preparation and law school admissions advice from PowerScore Test Preparation.

 Administrator
PowerScore Staff
  • PowerScore Staff
  • Posts: 8917
  • Joined: Feb 02, 2011
|
#40814
Complete Question Explanation
(The complete setup for this game can be found here: lsat/viewtopic.php?t=13302)

The correct answer choice is (C)

If there are at least two green balls, then either the 2-3-1 or 3-2-1 distribution is in effect. However, since the question stem also establishes that two green balls are in boxes 5 and 6, in order to adhere to the second rule there must be a third green ball lower in the stack. Thus, the 3-2-1 distribution is the sole possible distribution in this question. This leaves only two possible solutions:

PT63_Game_#4_#19_diagram 1.png
Accordingly, as shown in solution #1, answer choice (C) is possible, and therefore (C) is correct.
You do not have the required permissions to view the files attached to this post.
 elewis10
  • Posts: 21
  • Joined: Sep 02, 2017
|
#39137
How is it possible to have green balls in 5 and 6?? I thought red had to be above green in all cases??
User avatar
 Dave Killoran
PowerScore Staff
  • PowerScore Staff
  • Posts: 5852
  • Joined: Mar 25, 2011
|
#39144
Hi E,

Good question! The key here is the wording of the second rule, which states, "There is a box containing a green ball that is lower in the stack than any box that contains a red ball." (bold italics added for emphasis). So, what you are seeing there is that it doesn't say all the boxes containing green balls are lower than the boxes with red balls, but that there is at least one (that's the "a box" reference) that is lower than any box with a red ball. In other words, there's at least one box with a green ball that's going to be pretty low in the stack. As long as you have that, then you can also have green balls above red balls, as would be the case here.

The nice thing about this question stem being placed relatively early is that it immediately let's you know if you misinterpreted this rule, because if you read it as "all green are below red," then this stem seems to reference an impossible scenario. If that should ever happen again, don't question the stem, but instead go back to the scenario and rules and re-read them to see if something is different than you thought it was.

This is really tricky wording on their part, and a great example of why you have to read carefully and not just assume that the rule wordings are standard (which in this case would normally be the way you interpreted it).

Please let me know if that helps clear this one up. Thanks!
 plzhelpme101
  • Posts: 5
  • Joined: Oct 29, 2020
|
#80702
Administrator wrote:Complete Question Explanation
(The complete setup for this game can be found here: https://forum.powerscore.com/lsat/viewtopic.php?t=13302)

The correct answer choice is (C)

If there are at least two green balls, then either the 2-3-1 or 3-2-1 distribution is in effect. However, since the question stem also establishes that two green balls are in boxes 5 and 6, in order to adhere to the second rule there must be a third green ball lower in the stack. Thus, the 3-2-1 distribution is the sole possible distribution in this question. This leaves only two possible solutions:

PT63_Game_#4_#19_diagram 1.png
Accordingly, as shown in solution #1, answer choice (C) is possible, and therefore (C) is correct.
How do you have a red ball in box 4? Rule number 3 specifically prohibits this does it not? Box 4 would have to be white, would it not?
 Paul Marsh
PowerScore Staff
  • PowerScore Staff
  • Posts: 290
  • Joined: Oct 15, 2019
|
#80853
Hi plzhelpme! Box 4 certainly could be white, but it doesn't have to be. Our third rule just tells us that there needs to be one white ball that comes right below some green ball. Since we have 3 green balls in play here, we can satisfy our third rule by putting the white ball below any of them. One way that can happen is by putting our 3rd green ball in box 2, and our white ball in box 1. Check out diagram #1 in the post at the top of this thread, which explicitly draws out that possibility. Run through that diagram #1 and see how it satisfies all of our initial rules as well as our new Local rule.

Hope that helps! Please follow up if you still have any questions.

Get the most out of your LSAT Prep Plus subscription.

Analyze and track your performance with our Testing and Analytics Package.